Menu Close

Author: Tinku Tara

If-z-1-z-2-and-z-3-are-distinct-complex-numbers-such-that-z-1-z-2-z-3-1-and-z-1-2-z-2-z-3-z-2-2-z-3-z-1-z-3-2-z-1-z-2-1-then-the-value-o

Question Number 139700 by EnterUsername last updated on 30/Apr/21 $$\mathrm{If}\:{z}_{\mathrm{1}} ,\:{z}_{\mathrm{2}} \:\mathrm{and}\:{z}_{\mathrm{3}} \:\mathrm{are}\:\mathrm{distinct}\:\mathrm{complex}\:\mathrm{numbers}\:\mathrm{such} \\ $$$$\mathrm{that}\:\mid{z}_{\mathrm{1}} \mid=\mid{z}_{\mathrm{2}} \mid=\mid{z}_{\mathrm{3}} \mid=\mathrm{1}\:\mathrm{and} \\ $$$$\:\:\:\:\:\:\:\:\:\:\:\:\:\:\:\:\:\:\:\:\:\:\:\:\:\:\:\:\:\frac{{z}_{\mathrm{1}} ^{\mathrm{2}} }{{z}_{\mathrm{2}} {z}_{\mathrm{3}} }+\frac{{z}_{\mathrm{2}} ^{\mathrm{2}}…

is-it-always-satisfying-A-lim-n-f-n-x-dx-B-lim-n-f-n-x-dx-A-B-please-show-counter-example-checking-1-f-n-x-x-n-x-0-1-A-lim-n-0-1-x-n-dx-lim-n-1-n-1-x-n-1-0-B-0-1-

Question Number 8628 by sou1618 last updated on 18/Oct/16 $${is}\:{it}\:{always}\:{satisfying}? \\ $$$$\boldsymbol{{A}}=\mathrm{lim}\left[{n}\rightarrow\infty\right]\int{f}\left({n},{x}\right){dx} \\ $$$$\boldsymbol{{B}}=\int\mathrm{lim}\left[{n}\rightarrow\infty\right]{f}\left({n},{x}\right){dx} \\ $$$${A}={B}?? \\ $$$${please}\:{show}\:{counter}\:{example} \\ $$$$ \\ $$$$ \\ $$$${checking} \\…

In-an-entrance-examination-each-question-has-four-possible-answers-A-candidate-answers-three-question-by-guessing-a-find-the-probability-that-all-three-questions-are-wrong-b-find-the-probabili

Question Number 139697 by otchereabdullai@gmail.com last updated on 30/Apr/21 $$\mathrm{In}\:\mathrm{an}\:\mathrm{entrance}\:\mathrm{examination}\:\mathrm{each}\: \\ $$$$\mathrm{question}\:\mathrm{has}\:\mathrm{four}\:\mathrm{possible}\:\mathrm{answers}. \\ $$$$\mathrm{A}\:\mathrm{candidate}\:\mathrm{answers}\:\mathrm{three}\:\mathrm{question} \\ $$$$\mathrm{by}\:\mathrm{guessing}\:. \\ $$$$\mathrm{a}.\:\mathrm{find}\:\mathrm{the}\:\mathrm{probability}\:\mathrm{that}\:\mathrm{all}\:\mathrm{three}\: \\ $$$$\mathrm{questions}\:\mathrm{are}\:\mathrm{wrong}. \\ $$$$\mathrm{b}.\:\mathrm{find}\:\mathrm{the}\:\mathrm{probability}\:\mathrm{that}\:\mathrm{all}\:\mathrm{the}\: \\ $$$$\mathrm{questions}\:\mathrm{are}\:\mathrm{correct}. \\…

Question-139696

Question Number 139696 by mnjuly1970 last updated on 30/Apr/21 Answered by Dwaipayan Shikari last updated on 30/Apr/21 $$\vartheta\left(\theta\right)=\underset{{n}=\mathrm{1}} {\overset{\infty} {\sum}}\frac{\left(−\mathrm{1}\right)^{{n}+\mathrm{1}} {sin}\left({n}\theta\right)}{{n}\:} \\ $$$$\vartheta\left(\theta\right)=\frac{\mathrm{1}}{\mathrm{2}{i}}\underset{{n}=\mathrm{1}} {\overset{\infty} {\sum}}\frac{\left(−\mathrm{1}\right)^{{n}+\mathrm{1}}…

x-2-4x-x-3-

Question Number 139695 by john_santu last updated on 30/Apr/21 $$\:\mid{x}^{\mathrm{2}} −\mathrm{4}{x}\mid\:+\mid{x}\mid\:\geqslant\:\mathrm{3}\: \\ $$ Answered by MJS_new last updated on 30/Apr/21 $$\mid{x}\left({x}−\mathrm{4}\right)\mid+\mid{x}\mid=\mathrm{3} \\ $$$$\mathrm{squaring} \\ $$$${x}^{\mathrm{2}}…